Anda di halaman 1dari 54

February 2011

MEE Questions
and Analyses

Copyright 2011 by the National Conference of Bar Examiners.

All rights reserved.

February 2011 MEE Questions and Analyses

Contents

Preface............................................................................................................................................ iii

Description of the MEE ................................................................................................................. iii

Instructions..................................................................................................................................... iv

FEBRUARY 2011 QUESTIONS

Trusts Question ................................................................................................................................3

Evidence Question ...........................................................................................................................4

Secured Transactions Question........................................................................................................6

Torts Question..................................................................................................................................7

Family Law Question.......................................................................................................................8

Contracts Question...........................................................................................................................9

Corporations Question ...................................................................................................................10

Federal Civil Procedure Question..................................................................................................11

Decedents Estates Question..........................................................................................................12

FEBRUARY 2011 ANALYSES

Trusts Analysis...............................................................................................................................15

Evidence Analysis..........................................................................................................................18

Secured Transactions Analysis ......................................................................................................21

Torts Analysis ................................................................................................................................24

Family Law Analysis .....................................................................................................................28

i
Contents

Contracts Analysis .........................................................................................................................32

Corporations Analysis....................................................................................................................35

Federal Civil Procedure Analysis ..................................................................................................38

Decedents Estates Analysis ..........................................................................................................42

ii
Preface

The Multistate Essay Examination (MEE) is developed by the National Conference of Bar
Examiners (NCBE). This publication includes the questions and analyses from the February
2011 MEE. Each test includes nine questions; user jurisdictions may elect which of the nine
questions they wish to use. In the actual test, the questions are simply numbered rather than being
identified by area of law. The instructions for the test appear on page iv. For more information, see
the MEE Information Booklet, available on the NCBE website at www.ncbex.org.

The model analyses for the MEE are illustrative of the discussions that might appear in excellent
answers to the questions. They are provided to the user jurisdictions to assist graders in grading
the examination. They address all the legal and factual issues the drafters intended to raise in the
questions. Examinees need not present their responses in the same way or cover all the points
discussed in the model analyses to receive good grades.

The topics covered by each question are listed on the first page of its accompanying analysis,
followed by roman numerals that refer to the MEE subject matter outline for that topic. For
example, the Federal Civil Procedure question on the February 2011 MEE tested the following
areas from the Federal Civil Procedure outline: I.A.1., jurisdiction and venuesubject matter
jurisdictionfederal courts, and IV.A. & C., pretrial procedurespleadings and motions &
joinder of parties and claims (including class actions). Subject matter outlines are included in the
MEE Information Booklet.

Description of the MEE

The MEE is a series of essay questions, any of which a jurisdiction may select to include as a part
of its bar examination. It is administered by participating jurisdictions on the Tuesday before the
last Wednesday in February and July of each year. Examinees are expected to spend
approximately 30 minutes answering each MEE question administered. The areas of law that
may be covered by the questions on any MEE are Business Associations (Agency and
Partnership; Corporations and Limited Liability Companies), Conflict of Laws, Constitutional
Law, Contracts, Criminal Law and Procedure, Evidence, Family Law, Federal Civil Procedure,
Real Property, Torts, Trusts and Estates (Decedents Estates; Trusts and Future Interests), and
Uniform Commercial Code (Negotiable Instruments [Commercial Paper]; Secured Transac-
tions). Some questions may include issues in more than one area of law.

The purpose of the MEE is to test the examinees ability to (1) identify legal issues raised by a
hypothetical factual situation; (2) separate material which is relevant from that which is not; (3)
present a reasoned analysis of the relevant issues in a clear, concise, and well-organized
composition; and (4) demonstrate an understanding of the fundamental legal principles relevant
to the probable solution of the issues raised by the factual situation. The primary distinction
between the MEE and the Multistate Bar Examination (MBE) is that the MEE requires the
examinee to demonstrate an ability to communicate effectively in writing.

iii
Instructions
The back cover of each test form contains the following instructions:

Do not break the seal on this booklet until you are told to begin.

Each question is designed to be answered in 30 minutes. There will be no break once the
formal testing session begins. You may answer the questions in any order you wish. Do
not answer more than one question in each answer booklet. If you make a mistake or
wish to revise your answer, simply draw a line through the material you wish to delete.

If you are using a laptop computer to answer the questions, your jurisdiction will provide
you with specific instructions to follow.

Read each fact situation very carefully and do not assume facts that are not given in the
question. Do not assume that each question covers only a single area of the law; some of
the questions may cover more than one of the areas you are responsible for knowing.

Demonstrate your ability to reason and analyze. Each of your answers should show an
understanding of the facts, a recognition of the issues included, a knowledge of the
applicable principles of law, and the reasoning by which you arrive at your conclusion.
The value of your answer depends not as much upon your conclusions as upon the
presence and quality of the elements mentioned above.

Clarity and conciseness are important, but make your answer complete. Do not volunteer
irrelevant or immaterial information.

Your jurisdiction may instruct you to answer MEE questions according to the law of the
jurisdiction. Absent such an instruction, you should answer the questions by applying
fundamental legal principles rather than local case or local statutory law.

iv
February 2011
Questions

Trusts Question

In 1994, Testator died, survived by her sons, Ron and Sam, and Rons son, Peter. Under
Testators duly probated will, Testator created a trust and provided that

Trustee shall distribute to Ron, for the duration of his life, as much of the income and
principal from the trust as Trustee, in her uncontrolled discretion, deems advisable. At
Rons death, any remaining trust funds shall be distributed to my grandchildren, with the
children of any deceased grandchild taking the deceased parents share.

In another section of her will, Testator bequeathed property to her son Sam.

In 2006, Ron adopted Carol, who was then 14 years old.

On January 2, 2008, Ron borrowed $50,000 from a friend and gave the friend a note for that
amount payable in five annual installments of $10,000 plus interest.

Between 1994 and 2009, Trustee distributed all the trust income (which averaged $15,000 per
year) to Ron.

In 2009, Peter, Rons son, died in an automobile accident survived by his wife, Ginny, but by no
descendants. Under Peters valid will, his entire estate passed to Ginny.

On January 2, 2010, Ron failed to make the required payment to his friend on the $50,000 note.
Rons friend then demanded that Trustee distribute that years trust income to him instead of to
Ron until the $10,000 plus interest due that year was fully paid. At that point, Trustee stopped
paying trust income to Ron and refused to make payments to Rons friend. Instead, Trustee
began to accumulate trust income.

In late 2010, Ron died. He was survived by his adopted daughter, Carol; his daughter-in-law,
Ginny; his brother, Sam; and Sams wife. Sam has no children, but he and his wife plan to have a
child in the near future.

1. Was Rons friend entitled to any of the trust income earned during 2010? Explain.

2. At Rons death, to whom should the trust principal be distributed? Explain.

3
Evidence Question

On May 5, at 2 p.m. in City Park, Victim was hit from behind and temporarily knocked
unconscious. Upon regaining consciousness moments later, Victim discovered that his bag
containing valuables had been stolen.

While investigating the crime later that day, Police Officer interviewed Witness. Witness told
Police Officer that she had seen the robbery of Victim and had recognized Defendant, a resident
of the neighborhood, as the perpetrator. Witness also told Police Officer that Defendant had a
reputation in the neighborhood for violence, that everyone was afraid of him, and that she
shouldnt be talking to the police at all. Nevertheless, Witness agreed to accompany Police
Officer to police headquarters, where she looked at photographs of suspects and signed a written
statement. The statement read, I was walking in City Park on May 5, at 2 p.m., when I saw
Defendant. I saw Defendant attack Victim and then run away with Victims bag. I know
Defendant from the neighborhood and recognized Defendant as suspect number 1 on the 12-
person photograph display shown to me today by Police Officer.

Defendant was subsequently arrested and charged with robbery and assault.

At Defendants trial, Prosecutor called Witness to the stand. In response to questions from
Prosecutor, Witness testified that she had no memory of the incident. She stated that she did not
remember seeing anyone in City Park at the time of the alleged robbery. When Prosecutor asked
Witness whether her sudden memory loss was because she was afraid of Defendant, Witness said
that she had never seen Defendant before in her life and was not afraid of him because she did
not know him. When Witness was asked whether she had told Police Officer that Defendant had
robbed Victim, Witness denied ever making that statement.

Immediately after this testimony, Prosecutor offered Witnesss signed statement into evidence to
impeach Witnesss credibility and to prove that Defendant was in City Park and attacked Victim.
An authenticated copy of Witnesss statement was provided to Defense Counsel. Defense
Counsel raised no constitutional challenges to Witnesss identification of Defendant at police
headquarters. However, Defense Counsel objected to Prosecutor questioning Witness about the
statement and to admission of the copy of the statement. The judge sustained both objections.

After the prosecution had rested, Defense Counsel called Buddy to the stand. Buddy testified that
he had never met Defendant. He also testified that some of his friends had recently met
Defendant a few times, and that they think that Defendant is an honest and gentle person who
would never hurt anyone. Prosecutor objected to this testimony. The judge sustained the
objection and excluded Buddys testimony.

The rules of evidence in this jurisdiction are identical to the Federal Rules of Evidence.

1. Should the judge have permitted Prosecutor to question Witness about Witnesss written
statement and admitted the copy of the statement to impeach Witnesss credibility?
Explain.

4
Evidence Question

2. Should the judge have admitted Witnesss written statement to prove that Defendant was
in City Park and attacked Victim? Explain.

3. Should the judge have admitted Buddys testimony to prove Defendants character for
honesty and gentleness? Explain.

5
Secured Transactions Question

Astronomy Corporation (Astronomy) sells expensive telescopes to home stargazers. Astronomy


has a long-term financing arrangement pursuant to which it borrows money from Bank. In a
signed writing, Astronomy granted Bank a security interest in all its present and future inventory
to secure its obligations to Bank under the financing arrangement. Bank filed a properly
completed financing statement reflecting this transaction. The financing statement lists
Astronomy as the debtor and Bank as the secured party. The financing statement indicates that
the collateral is inventory.

Astronomy sells telescopes to some of its customers on credit. For a credit sale, Astronomy
requires the customer to sign an agreement granting Astronomy a security interest in the
purchased item to secure the customers obligation to pay the balance of the purchase price.

Six months ago, Johnson, an amateur stargazer, went to Astronomys showroom, saw a $3,000
telescope that he liked, and bought it on credit from Astronomy. Johnson paid $500 in cash and
agreed to pay the $2,500 balance in installment payments of $100 per month for the next 25
months, interest free. Consistent with Astronomys policy for credit sales, Johnson signed an
agreement granting Astronomy a security interest in the telescope to secure Johnsons obligation
to pay the balance of the purchase price. Astronomy did not file a financing statement with
respect to this transaction. At the time of the sale of the telescope to Johnson, Johnson was
unaware of the financial arrangement between Astronomy and Bank.

One month ago, Johnson sold the telescope for $2,700 in cash to his neighbor, Smith, another
amateur stargazer. Smith had no knowledge of any interest of Bank or Astronomy in the
telescope. Johnson then left the country without paying the remaining $2,000 owed to
Astronomy and cannot be located.

One week ago, Astronomy defaulted on its obligations to Bank.

Both Bank and Astronomy have discovered that Johnson sold the telescope to Smith. Bank and
Astronomy each have demanded that Smith surrender the telescope on the grounds that it is
collateral for obligations owed to them.

1. Does Bank have a security interest in the telescope that is enforceable against Smith?
Explain.

2. Does Astronomy have a security interest in the telescope that is enforceable against
Smith? Explain.

6
Torts Question

After recent terrorist threats, Metro Opera (Metro) decided to place metal detectors in its lobby.
Metro also marked off an area just beyond the metal detectors in which to search patrons who
failed the metal-detector test. Metro posted a sign near the entrance that read: Warning! No
metal objects allowed inside. All entrants are screened and may be searched.

Claimant and Friend saw the warning sign as they entered Metro. After entering, they observed
several patrons being frisked. Claimant said to Friend, Im certainly not going to allow anyone
to touch me!

Claimant then walked through the metal detector, which buzzed. Without asking Claimants
permission, Inspector, a Metro employee, approached Claimant from behind and began to frisk
Claimant. Claimant leaped away from Inspector and snarled, Leave me alone! Guard, another
Metro employee, then used a stun device, which administers a painful electric shock, to subdue
Claimant.

Unfortunately, the stun device, manufactured by Alertco, malfunctioned and produced a shock
considerably more severe than that described in Alertcos product specifications. The shock
caused minor physical injuries and triggered a severe depressive reaction that necessitated
Claimants hospitalization. Claimant had a history of depression but was in good mental health at
the time of the shock. Claimant was the first person who had ever experienced a depressive
reaction to the Alertco device.

The Alertco device malfunctioned because it was incorrectly assembled at the factory and
therefore did not meet Alertcos specifications. Alertcos assembly-inspection system exceeds
industry standards, and it is widely recognized as the best in the industry. Nonetheless, it did not
detect the assembly mistake in the device that injured Claimant.

Claimant has filed two tort actions seeking damages for her physical and psychological injuries:
(1) Claimant sued Metro, claiming that both the frisk and the use of the stun device were
actionable batteries, and (2) Claimant brought a strict products liability action against Alertco.

Metro has conceded that the actions of Inspector and Guard were within the scope of their
employment. Metro had instructed its employees to ask permission before frisking patrons, but
on the day Claimant was frisked, a supervisor told employees to frisk without asking permission
in order to speed up the entrance process.

1. Can Claimant establish a prima facie case of battery against Metro for (a) the use of the
stun device and (b) the frisk? Explain.

2. Does Metro have a viable defense to either battery claim? Explain.

3. Can Claimant establish the elements of a strict products liability claim against Alertco
based on the malfunction of the device? Explain.

4. Assuming that Claimant establishes either Metros or Alertcos liability, can Claimant
recover for her depressive reaction to the stun device? Explain.
7
Family Law Question

Husband and Wife married 12 years ago. Two years later, Wife gave birth to Child. Both
Husband and Wife are employed, and each earns approximately $80,000 per year.

Four months ago, Husband and Wife decided to divorce and entered into a written separation
agreement drafted by their respective attorneys. Under this agreement, Wife obtained sole title to
assets worth $175,000 and Husband obtained sole title to assets worth $125,000. All assets were
acquired during the marriage with employment income; there were no other assets. The
separation agreement provided that Wife would have sole custody of Child. It required Husband
to pay to Wife $500 per month in spousal support until her death or remarriage and $400 per
month in child support until Child reaches the age of 18.

At the time he signed the separation agreement, Husband was living with Fiancee,
a woman with
two teenage children. Indeed, his planned marriage to Fiancee was the primary reason for
Husbands willingness to sign the separation agreement.

Three months ago, Child was injured in an automobile accident. As a result of blood tests
performed following the accident, Husband discovered that he is not Childs biological parent.

Two months ago, at a hearing in the Husband-Wife divorce action, Husband petitioned the trial
court to invalidate the separation agreement based on unconscionability and fraud. The trial court
refused and entered a divorce decree incorporating the terms of the separation agreement.

After entry of the divorce judgment, Husband and Fiancee got married. Husband then filed a
motion to modify the divorce decree to

(a) grant him an equal share of the marital assets,


(b) award Wife no more than $200 per month in spousal support so that Husband could
meet the needs of [his] new family, and
(c) eliminate his child-support obligation based on Husbands nonpaternity of Child.

The trial court denied Husbands motion to modify the divorce decree.

1. Did the trial court err in denying Husbands petition to invalidate the separation
agreement on the basis of unconscionability and fraud? Explain.

2. Did the trial court err in denying Husbands motion to modify the divorce decree
according to each of the terms set forth in his motion? Explain.

8
Contracts Question

Designer and Retailer entered into a legally binding contract for Designer to maintain Retailers
website. Under the terms of the written contract, Retailer was to pay Designer $20,000 per year.
Retailer made timely payments for two years.

Eight months before the third years payment was due, Designer learned of an investment
opportunity. Designer called Retailer and said, I need cash quickly to make an investment that
will enable me to make a $35,000 profit. I know that you owe me $20,000, but if you promise
now to pay me $15,000 in cash by the 25th of this month, I will accept that payment as satisfying
your obligation under our contract for this year.

Retailer responded, Thanks. Thats a good deal. I dont have the cash to pay you now. Ill do it
if I can get a loan.

That will be great, responded Designer.

Because Designer assumed that Retailer would provide the cash Designer needed, Designer did
not try to raise the cash from another source.

Retailer, however, was busy with other matters. He visited two banks and picked up loan
applications, but he never bothered to submit them. Retailer did not take any other action to
obtain a loan before the 25th of the month had passed.

When the 25th of the month passed without payment from Retailer, Designer telephoned
Retailer. The moment that Retailer heard Designers voice saying Hello, Retailer quickly said,
Sorry, but I cant take you up on your offer to accept early payment.

Designer was shocked and angered. He had counted on that money. He can prove that he would
have gained $35,000 had he been able to make the planned investment.

Designer has sued Retailer for actual damages plus punitive damages.

1. Is Retailer liable for breach of contract? Explain.

2. Assuming that Retailer is liable, can Designer recover his actual damages from Retailer?
Explain.

3. Assuming that Retailer is liable, can Designer recover punitive damages? Explain.

9
Corporations Question

On September 1, Adam, Baker, and Clark formed a shoe manufacturing business called Delta
Incorporated (Delta). Each was to be a shareholder. Adam was named president of Delta.

Adam agreed to prepare and file articles of incorporation and bylaws for Delta, in accordance
with the states corporation statute, which is identical to the Model Business Corporation Act
(1984, with 2000 amendments). Adam, Baker, and Clark agreed to include a provision in Deltas
articles of incorporation stating that the corporations existence would begin on September 1.

On October 1, Adam, acting on behalf of Delta, entered into a contract with Mega Stores
Corporation (Mega) pursuant to which Mega was to purchase shoes from Delta for $3,000.
Following delivery of the shoes and after Mega had paid in full, Mega discovered that the shoes
did not conform to the contract specifications and returned the shoes to Delta. It is undisputed
that Delta owes Mega the $3,000 purchase price.

On October 15, Baker learned that Deltas articles of incorporation had not been filed.

On November 1, Adam, acting on behalf of Delta, entered into a contract with Sole Source, Inc.
(Sole), a supplier of shoe soles, pursuant to which Delta purchased shoe soles from Sole for
$100,000. The soles were delivered to Delta, and it is uncontested that Delta owes Sole the
$100,000 purchase price. Adam learned of the opportunity to contract with Sole from Baker,
who had worked with Sole in the past. Baker helped Adam negotiate the contract with Sole.

On November 15, Adam filed Deltas articles of incorporation with the appropriate state official.

When Delta did not pay either Mega or Sole the amounts it owed them, each company sued
Delta, Adam, Baker, and Clark for the amounts owed.

At all times, Clark believed that Deltas articles of incorporation had been filed.

1. When did Deltas corporate existence begin? Explain.

2. Is Adam, Baker, or Clark personally liable on the Mega contract? Explain as to each.

3. Is Adam, Baker, or Clark personally liable on the Sole contract? Explain as to each.

10
Federal Civil Procedure Question

Plaintiff, a citizen of State B, was vacationing in State A, where he visited the O.K. Bar. While
he was at the bar, Plaintiff was attacked and seriously beaten by Dave, a regular bar patron and a
citizen of State A. Bartender, a citizen of State A, attempted to stop the attack and was also
injured by Dave.

Plaintiff sued Dave and Bartender in the United States District Court for the District of State A,
properly invoking the courts diversity jurisdiction. Plaintiffs complaint states a state law battery
claim against Dave, seeking damages from Dave in excess of $75,000. Plaintiffs complaint also
states a claim against Bartender based on Bartenders alleged negligence in serving alcohol to
Dave after Dave became visibly intoxicated and belligerent. Plaintiffs complaint seeks damages
from Bartender in excess of $75,000. Plaintiffs damages claims are reasonable in light of the
injuries Plaintiff suffered in the attack.

Dave was personally served with the summons and complaint. However, the process server
could not find Bartender. He therefore taped the summons and complaint to the front door of the
O.K. Bar, where Bartender found them the next day.

Bartender made a timely motion to dismiss Plaintiffs complaint for failure to state a cause of
action. When that motion was denied by the district court judge, Bartender filed a second motion
to dismiss for insufficiency of service of process. The judge also denied that motion.

Bartender then filed an answer to the complaint, denying liability. The answer also stated a state
law claim for battery against Dave, seeking $20,000 damages for the injuries Bartender suffered
when he tried to stop Daves attack on Plaintiff.

Dave has moved to dismiss Bartenders cross-claim on the grounds of improper joinder and lack
of subject-matter jurisdiction.

1. Did the United States District Court for the District of State A properly deny Bartenders
motion to dismiss for insufficiency of service of process? Explain.

2. Do the Federal Rules of Civil Procedure permit Bartender to join a claim for battery
against Dave in Bartenders answer to Plaintiffs complaint? Explain.

3. Assuming that the Federal Rules of Civil Procedure permit Bartender to join his state law
claim against Dave, does the United States District Court for the District of State A have
subject-matter jurisdiction over that claim? Explain.

11
Decedents Estates Question

Two years ago, Testator purchased a $50,000 life insurance policy and named Niece as
beneficiary.

One year ago, Testator invited three friends to dinner. After dessert had been served, Testator
brought a handwritten document to the table and stated, This is my will. I would like each of
you to witness it. Testator then signed and dated the document. The three friends watched
Testator sign her name, and immediately thereafter, they signed their names below Testators
name.

One month ago, Testator died. Testator was survived by Niece, Cousin, and Son. Son is
Testators child from her first marriage. Testators second husband, Husband, died six months
before Testator. Husbands daughter from a prior marriage also survived Testator.

The handwritten document that Testator signed and that the three friends witnessed was found in
Testators desk. Its dispositive provisions provide in their entirety:

I, Testator, hereby make my Last Will and Testament.


I give my life insurance proceeds to Cousin.
I give the items listed in a memorandum to be found in my safe-deposit box to Niece.
I give $25,000 each to Church, Library, and School.
I give $40,000 to Husband.
I give the remainder of my assets to Son.

At Testators death, she owned the following assets:

1. The $50,000 life insurance policy, payable on Testators death to Niece


2. Jewelry worth $15,000
3. A bank account with a balance of $60,000

The jewelry was found in Testators safe-deposit box with a handwritten memorandum signed
and dated by Testator the day before she signed her will. The memorandum lists each piece of
jewelry and states, I want Niece to have all the jewelry here.

The terms of Testators life insurance contract provide that the beneficiary may be changed only
by submitting the change on the insurers change-of-beneficiary form to the insurance company.

State law explicitly disallows all holographic wills and codicils. To be valid, a will must be
acknowledged by the testator to the witnesses and signed by the testator in the presence of at
least two attesting witnesses, who shall sign their names below that of the testator within 30
days.

1. Is Testators will valid? Explain.

2. Assuming that Testators will is valid, who is entitled to


(a) Testators life insurance policy? Explain.
(b) Testators jewelry? Explain.
(c) Testators bank account? Explain.
12
February 2011
Analyses

Trusts Analysis

(Trusts I.D., E.1; II.A., B. & E.)

ANALYSIS

Legal Problems: (1) Does a creditor of a beneficiary of a discretionary trust have the
right to compel the trustee to make payments to the creditor?

(2) When an income beneficiary of a testamentary trust adopts a child


after the testators death, does the adopted child qualify as a
grandchild for purposes of sharing in a trust remainder?

(3) In a class gift to grandchildren following the death of a life tenant,


does the class close upon the life tenants death when there is a
grandchild eligible to take, even though more grandchildren could
be born after that date?

(4) When a trust instrument creates a remainder in grandchildren,


with the children of any deceased grandchild taking the deceased
parents share, does any interest vest in the estate of a grandchild
who dies without issue before the remainder becomes possessory?

DISCUSSION

Summary

A creditor of the beneficiary of a discretionary trust may not compel the trustee to make
payments to the creditor when the beneficiary cannot compel the trustee to make payments.
Thus, Rons friend had no claim against Trustee.
Adoption statutes generally treat adopted children equally with biological children for
purposes of inheritance and taking as class gift members under a will. Thus Carol, Rons adopted
daughter, will be entitled to share in the remainder interest in the trust even though she was not
adopted until after Testators death, because the class of grandchildren was still open at that time.
When a trust instrument creates a future interest in grandchildren, the class of
grandchildren closes when a grandchild becomes entitled to take, even if that means excluding
other potential (but yet unborn) grandchildren. Here, grandchildren became entitled to take at the
death of Testators son Ron, and the class closed at that time, thus excluding any future-born
grandchildren.
When a trust creates a future interest in grandchildren, with the children of any deceased
grandchild taking the deceased parents share, the estate of a grandchild who dies without issue
before the time for distribution succeeds to the grandchilds interest unless the jurisdiction has
adopted a statute akin to Uniform Probate Code 2-707, which creates a presumption that future
interests under the terms of a trust are contingent on survivorship until the distribution date.
[NOTE: There is no perpetuity issue in this question because Testator had a living
grandchild when she died (Peter) and thus the remainder interest is guaranteed to vest no later
than the death of Ron, who is a life in being.]

15
Trusts Analysis

Point One (30%)


Because the trust instrument conferred discretion on Trustee, Trustee was not obligated to make
payments to Ron or to Rons creditor.

The language of the trust instrument conferred uncontrolled discretion on Trustee to


make payments to, or withhold payments from, Ron. When, as here, a trustees discretion is
uncontrolled, there is no abuse of discretion if the trustee acts honestly and in a state of mind
contemplated by the trust creator. RESTATEMENT (THIRD) OF TRUSTS 50, cmt. c (2003). As a
result, Ron would be able to compel payments only if Ron could demonstrate that Trustee had
acted dishonestly or in a state of mind not contemplated by the trusts creator. There are no facts
here to support such a finding. See id.; SCOTT ON TRUSTS 128.3 (4th ed. 2001).
Because Rons creditor could have no greater rights in the trust than Ron had, the
creditor, Rons friend, could not compel Trustee to make payments to him. See RESTATEMENT
(THIRD) OF TRUSTS 56 (2003); SCOTT ON TRUSTS 155 (4th ed. 2001) (If the beneficiary
himself cannot compel the trustee to pay over any part of the trust fund, his creditors are in no
better position.). Moreover, even in the unlikely event that the friend could establish that
Trustees failure to make payments constituted an abuse of discretion, some modern authorities
hold that a creditor cannot assert the beneficiarys right to complain of abuse of discretion. See,
e.g., UNIF. TRUST CODE 504(b)(2) (providing that a creditor may not compel the trustee of a
discretionary trust to make a distribution, even when failure to make the distribution would
constitute an abuse of discretion).
[NOTE: This problem does not involve the law relating to the ability of creditors to come
after a beneficiarys interest when the instrument includes a spendthrift clause, as no such clause
appears in the instrument here.]
[NOTE: The following MEE jurisdictions have adopted the Uniform Trust Code:
Alabama, Arizona, Arkansas, District of Columbia, Maine, Missouri, Nebraska, New Hampshire,
New Mexico, North Dakota, Oregon, and Utah.]

Point Two (25%)


Carol, the adopted daughter of Ron, is entitled to a share of the remainder allocated to
grandchildren.

Section 2-705(a) of the Uniform Probate Code provides that [a]dopted individuals . . .
are included in class gifts . . . in accordance with the rules for intestate succession. Section 2-
114(b) of the Uniform Probate Code, which deals with intestate succession, provides that an
adopted individual is the child of his [or her] adopting parent or parents . . . . Thus, for purposes
of the trust, Carol is treated equally with a child born to Ron after Testators death. See also
RESTATEMENT (THIRD) OF PROPERTY (WILLS AND OTHER DONATIVE TRANSFERS) (Tentative
Draft No. 4 (2004) 14.5(2)(i) (In construing a class gift created by someone other than the
adoptive parent, the adopted child is treated as a child of the adoptive parent, but only if (i) the
adoption took place before the child reached the age of majority.).
Because it is clear that Carol should be treated as though she were Rons biological child,
the question is whether she qualifies to take the remainder even though she was not yet adopted
at Testators death. The answer is yes. The class of grandchildren closes when one person
becomes entitled to possession of the remainder interest. Id. 15.1. Because no person became
entitled to a share of the trust principal/remainder interest until Rons death, Carol is entitled to
share because she was adopted before that time.

16
Trusts Analysis

Point Three (20%)


Because at least one grandchild was entitled to take at the time of Rons death, the class of
grandchildren closed at that time, requiring distribution of the remainder at that time and
excluding future grandchildren from sharing in the remainder.

A class gift closes to future entrants when at least one class member is entitled to
distribution. RESTATEMENT (THIRD) OF PROPERTY (WILLS AND OTHER DONATIVE TRANSFERS)
Tentative Draft No. 4 15.1 (2004). In this case, because grandchildren were entitled to take at
Rons death, the class closed, and any children subsequently born to Sam were excluded from
sharing in the remainder.

Point Four (25%)


Ginny, the beneficiary of Peters will, is entitled to share in the remainder to grandchildren
unless the jurisdiction has adopted a statute akin to Uniform Probate Code 2-707.

When a trust creates a vested remainder in a person or member of a class and then
provides that the remainder should pass to that persons child if the remainder person
predeceases the life tenant, the remainder is divested only if the deceased remainder person has a
child. If, on the other hand, the remainder person dies childless, the remainder is not divested and
passes to the remainder persons estate. See, e.g., Swanson v. Swanson, 514 S.E.2d 822 (Ga.
1999); Matter of Krooss, 99 N.E.2d 222 (N.Y. 1951). Here, Peters remainder would have been
divested in favor of his child if he had been survived by a child, but because he was not survived
by a child, the remainder vested in his estate. And, because Peter left his entire estate to his wife,
Ginny, she succeeds to Peters interest in the remainder.
The result would be different if the jurisdiction has adopted 2-707 of the Uniform
Probate Code or a similar statute. Section 2-707(b), a controversial provision, makes future
interests under the terms of a trust contingent on the beneficiarys surviving to the distribution
date. Under UPC 2-707(b), then, Peters remainder was contingent on surviving Ron. Because
Peter did not survive Ron, his interest did not vest. As a result, he had no interest to pass to
Ginny.
Hence, at common law, Carol and Ginny would share the remainder equally; if UPC
2-707 or a similar statute is in force, only Carol would succeed to the entire remainder.
[NOTE: The following MEE jurisdictions have enacted 2-707 of the Uniform Probate
Code: Massachusetts, Michigan, Montana, North Dakota, Utah, and Iowa (modified).]

17

Evidence Analysis

(Evidence I.D.1.; II.C.; V.A.)

ANALYSIS

Legal Problems: (1) May a witness who testifies to a lack of memory at trial be
impeached with a prior inconsistent statement, and may extrinsic
evidence of the prior inconsistent statement be admitted?

(2) May a prior statement of identification be admitted?

(3) May a prior inconsistent statement made to the police be admitted


to prove the elements of the charged offenses?

(4) May a defense witnesss testimony about what his friends think of
a criminal defendant whom they met recently just a few times be
admitted to prove that the defendant is honest and gentle?

DISCUSSION

Summary

Witnesss statement to Police Officer contradicted her testimony at trial. This is a prior
inconsistent statement that can be used at trial to impeach her credibility. Because Witnesss
testimony that she did not remember the incident and did not recognize Defendant was
inconsistent with her written statement, because her lack of memory was likely feigned, and
because she was given an opportunity to explain the statement, the court should have permitted
Prosecutor to question Witness about her prior inconsistent statement and admitted the
authenticated copy of Witnesss statement to impeach Witness.
Part of Witnesss statementI was walking in City Park on May 5, at 2 p.m., when I
saw Defendant. I know Defendant from the neighborhood and recognized Defendant as suspect
number 1 on the 12-person photograph display shown to me today by Police Officershould
have been admitted as a statement of identification, which is not hearsay as defined in the
Federal Rules of Evidence. FED. R. EVID. 801(d)(1)(C). The remainder of Witnesss statement
I saw Defendant attack Victim and then run away with Victims bagwas properly excluded
because, although inconsistent, the statement was not made under oath subject to the penalty of
perjury at a trial, hearing, or other proceeding, or in a deposition. FED. R. EVID. 801(d)(1)(A).
Buddys testimony was properly excluded because even relevant character evidence must
be introduced in the form of proper opinion or reputation evidence. Buddy did not testify to his
own opinion, and Buddys knowledge of his friends opinions did not provide evidence of
Defendants reputation.

Point One (25%)


A prior inconsistent statement is ordinarily admissible for the purpose of impeaching the
credibility of a witness. Extrinsic evidence of the prior statement, such as the written statement

18
Evidence Analysis

itself, may be admitted when the witness has been given an opportunity to explain or deny the
statement.

It is generally permissible for a litigant to impeach the credibility of any witness,


including a witness called by that litigant. FED. R. EVID. 607. Moreover, it is common and proper
to impeach a witnesss credibility by showing that the witness has made prior statements
inconsistent with his [or her] testimony at trial. GRAHAM C. LILLY, AN INTRODUCTION TO THE
LAW OF EVIDENCE 8.5, at 414 (3d ed. 1996).
In order to introduce a prior out-of-court statement to impeach a witnesss credibility,
there must be an inconsistency between the prior out-of-court statement and the witnesss trial
testimony. The inconsistency must involve a relevant issue. If a witness testifies to a lack of
memory regarding a relevant issue contained in a prior out-of-court statement, the judge may
find that this testimony is inconsistent with the out-of-court statement on the same issue.
Moreover, if the judge finds that the lack of memory is feigned, the court is more likely to find
that the testimony and statement are inconsistent. See MICHAEL H. GRAHAM, HANDBOOK ON
FEDERAL EVIDENCE 613.2 (5th ed. 2001). Here, there is more than a claimed lack of present
memory concerning the facts. Witness testified that she had never seen Defendant in her life and
denied that she had told Police Officer that she saw Defendant rob Victim. Thus, her trial
testimony is inconsistent with her previous written statement. Her testimony is also strongly
suggestive of feigned memory loss.
Thus, the court should have allowed Prosecutor to question Witness about the entire
statement. The written statement may also be admitted as extrinsic evidence. Witness should also
have been given an opportunity to explain or deny all or part of the written statement, and the
statement should have been admitted to impeach Witnesss credibility by proving that she made
prior inconsistent statements. Admission of Witnesss statement for the limited purpose of
impeaching her trial testimony does not raise hearsay concerns because it is not admitted to
prove the truth of the matter asserted in the statement.
[NOTE: At Defense Counsels request, the judge should instruct the jury that the
prosecution cannot rely on Witnesss prior written statement, which was admitted solely to
impeach Witnesss credibility, to prove any of the elements of the charged offenses.]

Point Two (25%)


Part of Witnesss statementI was walking in City Park on May 5, at 2 p.m., when I saw
Defendant. I know Defendant from the neighborhood and recognized Defendant as suspect
number 1 on the 12-person photograph display shown to me today by Police Officershould
have been admitted as a statement of identification, which is not hearsay.

A prior statement of identification of a person made after perceiving the person is not
hearsay if the witness who made the statement testifies at trial and is subject to cross-
examination concerning the statement. FED. R. EVID. 801(d)(1)(C). The rationale for this rule is
that the opportunity to cross-examine the witness concerning the statement takes it outside the
scope of the hearsay rule. FED. R. EVID. 801(d)(1). For the statement to be admissible to prove
the truth of the matter asserted under this provision, it is not necessary that the speaker be able to
confirm the identification at trial or remember the identity of the person. If the declarant testifies
and is subject to cross-examination concerning the identification, the declarants lack of memory
at trial does not defeat admissibility and the requirements of Rule 801(d)(1)(C) are fulfilled.
United States v. Owens, 484 U.S. 554 (1988).

19
Evidence Analysis

In this case, Witnesss prior written statement included an identification of Defendant.


That identification was made by Witness based on Witnesss knowledge of Defendant from the
neighborhood, Witnesss direct observation of Defendant in City Park on May 5 at 2 p.m., and
Witnesss recognition of Defendants photograph. That prior statement of identification of
Defendant is excluded from the general definition of hearsay when, as here, Witness testifies and
is subject to cross-examination concerning the statement.

Point Three (20%)


The remainder of Witnesss prior inconsistent statement made to the policeI saw Defendant
attack Victim and then run away with Victims bagmay not be admitted to prove the elements
of the charged offenses because it is hearsay.

Under the Federal Rules of Evidence, certain prior inconsistent statements are not
hearsay. FED. R. EVID. 801(d)(1)(A). However, to be admissible to prove the truth of the matter
asserted, the prior inconsistent statement must be made under oath, subject to the penalty of
perjury at a trial, hearing, or other proceeding, or in a deposition. Id. Witnesss statement to the
police does not fit within this rule, nor does it fit within any hearsay exception.

Point Four (30%)


A criminal defendant may introduce evidence of his good character relating to a relevant
character trait. Character may be proved only through testimony of reputation or opinion.

Character evidence is generally inadmissible to prove that a person acted in conformity


with a particular character trait. However, a defendant in a criminal case is permitted to offer
evidence of a relevant character trait to prove that the defendant did not commit the charged
offense. FED. R. EVID. 404(a)(1). Because Defendant is charged with robbery and assault (both
crimes of violence), evidence that Defendant is a gentle person who would never hurt anyone
would be relevant. FED. R. EVID. 401. Evidence that Defendant is honest is a closer call.
Committing a robbery or an assault is not inconsistent with having a reputation for truth-telling,
if being honest means being a truth-teller. See United States v. Darland, 626 F.2d 1235 (5th
Cir. 1980). On the other hand, robbery is a form of theft, and theft is dishonest. So a court might
admit evidence of Defendants honesty, as well as evidence of his gentleness.
However, the Federal Rules of Evidence limit the ways in which a defendants character
traits may be proven. See FED. R. EVID. 405(a). In particular, proof may [only] be made by
testimony as to reputation or by testimony in the form of an opinion. Id. See also Michelson v.
United States, 335 U.S. 469 (1948).
Here, Buddy did not testify as to his own opinion or Defendants community reputation.
Rather, he said that a few of his friends think that Defendant is honest and gentle. Buddys
testimony about his friends opinions of Defendant does not qualify as evidence of Defendants
reputation in the community. Community includes circles of associates where one lives,
works, or regularly socializes. United States v. Oliver, 492 F.2d 943 (8th Cir. 1974); United
States v. Parker, 447 F.2d 826, 83031 (7th Cir. 1971). Buddys friends opinions do not qualify
as reputation testimony because the friends met Defendant recently and only a few times.

20
Secured Transactions Analysis

(Secured Transactions I.B.; II.D. & E.; III.B.; IV.B. & C.)

ANALYSIS

Legal Problems: (1)(a) Did Bank have a security interest in the telescope before it was
sold by Astronomy? Was the security interest perfected?

(1)(b) Did Banks security interest in the telescope continue after the
successive sales of the telescope to Johnson and Smith?

(2)(a) Did Astronomy have a security interest in the telescope before


Johnson sold it to Smith? Was the security interest perfected?

(2)(b) Did Astronomys security interest in the telescope continue to be


enforceable after Johnson sold the telescope to Smith?

DISCUSSION

Summary

Bank had a perfected security interest in Astronomys inventory. When one item of that
inventorythe telescopewas sold to Johnson, Johnson took the telescope free of Banks
security interest in it because Johnson was a buyer in ordinary course of business. Because
Johnson owned the telescope free of Banks security interest, Johnson transferred it to Smith free
of that security interest. Astronomy retained a security interest in the telescope when Johnson
bought it from Astronomy. The security interest was perfected even though Astronomy did not
file a financing statement because the security interest was a purchase-money security interest in
consumer goods. When Smith later bought the telescope from Johnson, he took free of
Astronomys security interest because of the protection afforded to buyers in consumer-to-
consumer transactions by Article 9 of the Uniform Commercial Code.

Point One(a) (20%)


Bank had a perfected security interest in the telescope before it was sold.

Because Astronomy held the telescope for sale, the telescope was part of Astronomys
inventory. See UCC 9-102(a)(48). Thus, it was covered by the security interest granted by
Astronomy to Bank. The security interest was enforceable and attached because the three criteria
in UCC 9-203(b) were fulfilled: value had been given (Bank loaned money to Astronomy),
Astronomy had rights in the telescope, and Astronomy had authenticated (signed or its electronic
equivalent) a security agreement containing a description of the collateral. Banks security
interest in the telescope was also perfected. Filing the statement satisfied the requirement in UCC
9-310 that a financing statement must be filed to perfect a nonpossessory security interest in
goods.

21
Secured Transactions Analysis

Point One(b) (30%)


Banks security interest in the telescope did not continue after the successive sales of the
telescope to Johnson and Smith.

As a general rule, a security interest continues after sale of the collateral. UCC 9-
315(a)(1). However, that general rule is subject to many exceptions, one of which applies here: a
buyer in ordinary course of business (BIOCOB) takes free of a security interest created by that
buyers seller. See UCC 9-320(a). A BIOCOB is a buyer who buys goods in good faith,
without knowledge that the sale violates the rights of another, from a person in the business of
selling goods of the kind, in the ordinary course of the sellers business. See UCC 1-201(b)(9)
(UCC 1-201(9) in states that have not enacted Revised Article 1). Johnson qualifies as a
BIOCOB. Johnson bought the telescope from a person (Astronomy) in the business of selling
goods of the kind in the ordinary course of Astronomys business, and there is no indication in
these facts that Johnson failed to act in good faith or had knowledge that the sale violated the
rights of another. Because Johnson was a BIOCOB, he took the telescope free of Banks security
interest, which was created by his seller, Astronomy.
Since Johnson owned the telescope free of that security interest, under general property
principles all of Johnsons property rights in the telescope were transferred to Smith when
Johnson sold the telescope to Smith. See also UCC 2-403(1). Thus, Smith acquired the
telescope free of Banks security interest. This is known in secured transactions law as the
shelter principle.

Point Two(a) (25%)


Before Johnson sold the telescope, Astronomy had a perfected security interest in it.

The security agreement between Johnson and Astronomy provided that Astronomy
retained a security interest in the telescope to secure Johnsons obligation to pay the remainder of
the purchase price. The security interest was enforceable and attached because the three criteria
in UCC 9-203(b) were fulfilled: value had been given (the telescope), Johnson had rights in the
telescope, and Johnson had authenticated (signed or its electronic equivalent) a security
agreement containing a description of the collateral.
Astronomys security interest in the telescope was also perfected, even though
Astronomy did not file a financing statement. This is because Astronomys security interest was
a purchase-money security interest (PMSI) in consumer goods. It was a PMSI because it was
retained by the seller of the telescope to secure the buyers obligation to pay the remainder of the
purchase price. See UCC 9-103. The telescope constituted consumer goods because it was
bought by Johnson for personal, family, or household purposes. UCC 9-309(1) provides that a
PMSI in consumer goods perfects automatically upon attachment; thus, it was not necessary for
Astronomy to file a financing statement to perfect its security interest.

Point Two(b) (25%)


Astronomys security interest in the telescope is not enforceable against Smith.

Prior to the sale from Johnson to Smith, Astronomy had a perfected security interest in
the telescope. (See Point Two(a).) Smith does not qualify as a BIOCOB because, inter alia,
Smith did not buy the telescope from someone in the business of selling goods of that kind.
(Johnson was not in the business of selling telescopes.) Thus, Smith did not take free of

22
Secured Transactions Analysis

Astronomys security interest pursuant to UCC 9-320(a). However, a buyer of consumer goods
takes free of a security interest in those goods if the buyer buys without knowledge of the
security interest; gives value; buys for personal, family, or household use; and receives the goods
before the filing of any financing statement covering them. See UCC 9-320(b). Smith fulfilled
these criteria. (Recall that Astronomy was not required to file a financing statement to perfect its
security interest because it was a purchase-money security interest.) Thus, Smith took the
telescope free of Astronomys security interest in it.
[NOTE: Some examinees may note that Astronomys rights against Johnson constitute
chattel paper in which Bank has a security interest as proceeds of its original collateral. This is
correct but does not affect the answer to the question posed because Smith is free of both
security interests.]

23

Torts Analysis

(Torts I.A., B.1.; III; IV)

ANALYSIS

Legal Problems: (1)(a) Did use of a stun device to shock Claimant establish a prima facie
case of battery?

(1)(b) Did frisking


Claimant establish a prima facie case of battery?

(2) Did Claimant consent to either the frisk or the stun-device shock?

(3) Is the manufacturing defect in the Alertco device actionable in


strict products liability?

(4) Can Claimant recover damages for her depressive reaction to the
malfunctioning stun device even though that depressive reaction
was unforeseeable?

DISCUSSION

Summary

A prima facie case of battery is established when the plaintiff shows that she suffered a
harmful or offensive contact and that the defendant intended to cause such a contact. Guards use
of a stun device against Claimant meets this test because Guard intentionally administered a
painful and physically incapacitating shock to Claimants body. However, it is unclear whether
Inspectors frisk of Claimants body satisfies this test, as such a frisk may not be sufficiently
offensive to constitute battery and Inspector may have been unaware that Claimant would be
offended by the frisk. Moreover, because conduct can manifest consent, Claimant may have
consented to the frisk when she walked through the metal detector. On the other hand, Claimant
did not manifest consent to being shocked with the stun device.
A manufacturer is strictly liable for manufacturing defects. Alertco is thus liable for the
defective stun device because the defect arose during the manufacturing process.
Because a tort defendant takes his victim as he finds him, Claimant may recover for her
unforeseeable depressive reaction to the stun device.

Point One(a) (20%)


A prima facie case of battery is established when the plaintiff shows that she suffered a harmful
or offensive contact and that the defendant intended to cause such a contact. Guards use of a
stun device against Claimant satisfies this test. Thus, because Guard was acting within the scope
of his employment, Claimant can establish a prima facie case against Metro.

An actor is subject to liability to another for battery if (a) he acts intending to cause a
harmful or offensive contact with the person and (b) a harmful or offensive contact results.
RESTATEMENT (SECOND) OF TORTS 13 (Battery: Harmful Contact) & 18 (Battery: Offensive

24
Torts Analysis

Contact) (1965). Intent, for purposes of establishing a battery, means either that the actor
desires to cause [the] consequences of his act or that he knows that the consequences are
certain, or substantially certain, to result from his act . . . . Id. 8A & cmt. b.
Guards use of a stun device to subdue Claimant meets these standards for an actionable
battery. First, the use of a stun device both incapacitates the target and causes the target to suffer
pain. Either effect constitutes bodily harm for purposes of establishing a prima facie case of
battery. See id. 15 (bodily harm includes any physical impairment of the condition of
anothers body, or physical pain or illness). Second, Guard certainly used the stun device for the
purpose of physically impairing Claimant. His goal was to make it easier to subdue Claimant by
causing physical impairment. Furthermore, although the facts do not say so explicitly, it is a
reasonable assumption that Guard was aware that stun devices cause pain when they are used
against someone. Thus, Guards actions amount to battery: Guard intended to, and did, cause a
harmful contact with Claimant. Because Guard was acting within the scope of his employment,
Claimant can establish a prima facie case against Metro.

Point One(b) (20%)


It is unclear whether Inspectors frisk of Claimant establishes a prima facie case of battery. It is
possible that Claimant will not be able to show intent to cause a harmful or offensive contact.

Inspectors frisk of Claimant was not harmful, but a contact can also constitute battery if
it is offensive. A bodily contact is offensive if it offends a reasonable sense of personal dignity.
RESTATEMENT (SECOND) OF TORTS 19 (1965). The facts make clear that Claimant found the
contact offensive, but this is not determinative. Claimant must additionally show that being
frisked offends a reasonable sense of dignity.
The frisk in this case was part of a routine screening process, and screening like this is
now frequently encountered in airports and other public places. The routine nature of such
screening may mean that it is no longer reasonable to be offended by it. On the other hand, the
fact that Metro generally required employees to ask permission before frisking opera patrons
suggests that frisking is often experienced by people as offensive, especially if it occurs without
permission. It is thus possible, but not certain, that a routine frisk would be viewed by a court as
a contact that offends a reasonable sense of personal dignity.
Even if the frisk were considered offensive, a prima facie case of battery based on the
frisk cannot be established unless Inspector is shown to have acted with intent to cause an
offensive contact. To establish intent, Claimant must show that Inspector had the purpose of
causing offense or knew he would cause offense by frisking Claimant. It is not enough to show
that Inspector may have been aware of a risk that some patrons would be offended by a frisk.
Batterys protection against offensive contact is intended to protect a persons dignitary
interest. That interest is protected only against intentional invasion. The actor is not liable for an
act which involves a risk, no matter how great and unreasonable, that it will cause . . . an
offensive contact. Id. 18, cmt. g (emphasis added).
Because opera patrons were warned of the possibility of being frisked, Inspector may
have assumed that they would not be offended by such a contact. On the other hand, Inspector
did not warn Claimant before initiating the frisk in question, and it is unclear in what manner
Inspector conducted that frisk. Claimant can establish a prima facie case if she can present some
facts suggesting that Inspector knew with substantial certainty that Claimants reasonable sense
of personal dignity would be offended by Inspectors decision to touch her without warning, or
that Inspector knew that the manner in which he touched Claimant would be offensive to her.

25
Torts Analysis

But, absent some facts suggesting Inspectors knowledge that the frisk would be offensive, the
mere fact that Inspector intentionally contacted Claimants person does not satisfy the
requirements for an actionable battery.
[NOTE: The examinees analysis is more important than his or her conclusion.]

Point Two (30%)


Because conduct can manifest consent, Claimant may have consented to being frisked. Claimant
did not manifest consent to being shocked with the stun device.

Consentwillingness in fact for conduct to occuris a defense to battery.


RESTATEMENT (SECOND) OF TORTS 892. Consent may be manifested by action or inaction and
need not be communicated to the actor. Thus, [i]f words or conduct are reasonably understood
by another to be intended as consent, they constitute apparent consent and are as effective as
consent in fact. Id.
Claimant did not intend to consent to Inspectors search of her person. Claimant told
Friend, Im certainly not going to allow anyone to touch me, and told Inspector, Leave me
alone!
However, a defendant is entitled to rely on objective manifestations of consent. Thus,
because Claimant read the warning sign, saw opera patrons being frisked, and still proceeded
through the metal detector, Metro can argue that its employee, Inspector, was entitled to infer
that Claimant had consented to being frisked herself. See OBrien v. Cunard Steamship Co., 28
N.E. 266, 274 (Mass. 1891) (finding that the plaintiff consented to vaccination when she stood in
line with other passengers and there was nothing in [her] . . . conduct . . . to indicate . . . that she
did not wish to obtain a [vaccination] card).
On the other hand, the fact that Metro usually required its employees to request
permission before frisking patrons suggests that it did not infer consent to frisking from a
patrons decision to proceed through the metal detector. A court thus might conclude that it was
unreasonable to infer Claimants consent to being frisked.
Metro has no possible argument that Claimant consented to being shocked with a stun
device. Claimant did not see the stun device in use, and there were no signs warning of its use.
There are no other defenses available to Metro. The privilege to exercise force in self-
defense or defense of others is unavailable because Claimants statement (Leave me alone!)
could not reasonably be interpreted as a threat. See RESTATEMENT (SECOND) OF TORTS 63 (An
actor is privileged to use reasonable force . . . to defend himself against unprivileged harmful or
offensive contact or other bodily harm which he reasonably believes that another is about to
inflict . . . .).

Point Three (15%)


A manufacturer is strictly liable for a defective product. Alertco is thus liable for the defective
stun device even though Claimant cannot show that Alertco was negligent.

Alertcos product contained a manufacturing defect; it failed to meet Alertcos own


specifications, and the defect occurred during the products manufacture. See RESTATEMENT
(THIRD) OF TORTS: PRODUCTS LIABILITY 2 (1998). In order to recover for injuries sustained
because of a manufacturing defect, a plaintiff need not show that the manufacturer was negligent.
The manufacturer is strictly liable whenever the product departs from its intended design even
though all possible care was exercised in the preparation and marketing of the product . . . . Id.

26

Torts Analysis

In this case, the evidence establishes that the defect occurred during the manufacturing process.
Thus, despite Alertcos excellent quality control, it is strictly liable to Claimant.

Point Four (15%)


Because a tort defendant takes his victim as he finds him, Claimant may recover for her
unforeseeable depressive reaction to the stun device.

When an actors tortious conduct causes harm to a person that, because of a preexisting
physical or mental condition or other characteristics of the person, is of a greater magnitude or
different type than might reasonably be expected, the actor is nevertheless subject to liability for
all such harm to the person. RESTATEMENT OF TORTS: LIABILITY FOR PHYSICAL HARM 31
(2010). Thus, the plaintiff with an eggshell skull who suffers injuries different from or greater
than those that a normal victim would suffer is entitled to recover fully for his injuries.
Claimants history of depression is a preexisting condition, like an eggshell skull. Thus,
despite the fact that a depressive reaction to the stun device was unforeseeable, Alertco is liable
for both Claimants physical and psychological injuries. See Bartolone v. Jeckovich, 481
N.Y.S.2d 545 (N.Y. App. Div. 1984); Steinhauser v. Hertz Corp., 421 F.2d 1169 (2d Cir. 1970).

27
Family Law Analysis

(Family Law III.D., F., G. & J.; V.B.3.)

ANALYSIS

Legal Problems: (1) On what basis may the property-division and support provisions of
a separation agreement be invalidated, and is there evidence to
support invalidation in this case?

(2) When, if ever, may the property-division provisions of a divorce


decree be modified?

(3) When, if ever, may the spousal- and child-support provisions of a


divorce decree be modified?

(4) May the nonpaternity of a mothers husband be established after


entry of a divorce judgment when the child is 10 years of age and
the husband is the only father the child has ever known?

DISCUSSION

Summary

Both the support and property-division provisions of a separation agreement may be


invalidated based on a finding of unconscionability or fraud. Here, there are insufficient facts to
support a finding of unconscionability with respect to the property-division or spousal-support
portions of the agreement. Given the variety of approaches currently applied by the states to
paternity misrepresentation and disestablishment, it is unclear whether the court erred in denying
Husbands petition to invalidate the child-support provisions of the separation agreement.
A spousal- or child-support order may be modified based on a change of circumstances
after the order was entered. However, there has been no change of circumstances in this case
because Husband knew or should have known of the economic needs of his intended wife and
stepchildren before signing the agreement, and there is no evidence that those needs have
changed. It is unclear whether Husband can obtain modification of his child-support obligation
based on his nonpaternity of Child.

Point One (40%)


A separation agreement may be invalidated, in whole or in part, based on unconscionability or
fraud. Here, there are insufficient facts to support a finding of unconscionability with respect to
the property-division or spousal-support portions of the agreement. Given the variety of
approaches currently applied by the states to paternity misrepresentation and disestablishment, it
is unclear whether the trial court erred in denying Husbands petition to invalidate the child-
support provisions of the separation agreement based on fraud.

All states authorize invalidation of a separation agreement, or a specific portion of such


an agreement, based on a finding of fraud or unconscionability. Here, there is no evidence of

28
Family Law Analysis

asset or income misrepresentation. And, although judicial interpretations of the unconscionability


test vary (compare Jameson v. Jameson, 239 N.W.2d 5 (S.D. 1976) with Williams v. Williams,
508 A.2d 985 (Md. 1986)), the facts do not appear to support an unconscionability finding with
respect to the property-division or spousal-support portions of the agreement. The parties were
represented by separate counsel, and there is no evidence of asset nondisclosure. Nor will the
unconscionability doctrine serve to relieve a spouse whose complaint is simply that he believes
he made a bad bargain. See Jameson, 239 N.W.2d at 7. It is not even clear that Husband made a
bad bargain. Although he did receive fewer assets than Wife, most states require equitable, not
equal, division of assets, and equity is determined through a complex assessment of contribution
and need-based factors. See HARRY A. KRAUSE ET AL., FAMILY LAW: CASES, COMMENTS, AND
QUESTIONS (6th ed. 2007) at 752. Moreover, the agreement requires Husband to pay only
$10,800 per year (13.5 percent of his total income) in combined child and spousal support. Many
state child-support guidelines would require Husband to pay this much or more in child support
alone. See id. at 89193 (reviewing support guidelines). Thus, it is highly unlikely that an
appellate court would find reversible error in the trial courts refusal to invalidate these portions
of the agreement based on unconscionability.
It is unclear whether an appellate court would find reversible error in the trial courts
refusal to invalidate the child-support portion of the agreement, however. It is now apparent that
Husband is not Childs biological parent. Arguably, Wifes failure to alert Husband to this
possibility was a form of misrepresentation which might support a finding of fraud. A few courts
have permitted tort actions against a wife who misrepresented her husbands paternity. See
Miller v. Miller, 956 P.2d 887 (Okla. 1998); G.A.W., III v. D.M.W., 596 N.W.2d 284 (Minn.
App. 1999). In a state with such case law, it is possible that an appellate court might conclude
that Wifes conduct provided a basis for invalidating the child-support provisions of the
agreement. See Point Three, infra.

Point Two (10%)


A divorce property-division award is not subject to modification.

Although a support award may be modified after a divorce decree has been entered, a
property-division award, whether it results from a judicial decision or a divorce settlement
agreement, may not be modified after a divorce decree has been entered. See KRAUSE ET AL.,
supra, at 991. This distinction rests on a fundamental difference between support and property
division. A support order requires payment, over what may be a long period of time, of post-
divorce income, while a property-division award divides assets accrued during the marriage
based on the equities at the time of divorce. Property division thus reflects an evaluation of the
past, and support determination reflects an evaluation of the future. Because the future is
unpredictable, courts are empowered to modify a support award to take account of changed
circumstances. Because the past can be ascertained, courts are not empowered to reconsider that
evaluation and modify a property-division award once it has been embodied in a final judgment
of divorce. Thus, the trial court did not err in refusing to modify the property-division provisions
of the divorce decree.

Point Three (25%)


A spousal-support award may be modified when the court finds that there has been a substantial
change in a partys circumstances. Because Husband knew of his future obligations to his new

29
Family Law Analysis

spouse and stepchildren at the time he signed the agreement, there is no change of circumstances
in this case.

In all states, a spousal-support award may be modified based on a change in a partys


circumstances. Most courts require that such a change be substantial. See HARRY D. KRAUSE &
DAVID D. MEYER, FAMILY LAW IN A NUTSHELL, 30305 (5th ed. 2007).
A court may take into account legal obligations to a new spouse and/or children in
determining whether there has been a change in circumstances that warrants modification of a
support award. See id. However, in this case, Husband does not have new adopted or biological
children to whom he owes support; he has stepchildren. In most states, a stepparent relationship
creates no legal support obligations. See id. at 17576.
More importantly, a change in circumstances that is anticipated may not serve as a basis
for modification of a support order. See M.L. Cross, Annotation, Change in Financial Condition
or Needs of Husband or Wife as Ground for Modification of Decree for Alimony or
Maintenance, 18 A.L.R.2d 10 3 (1951 & Supp.). Here, Husband knew about the support needs
of Fiancee and her children at the time he signed the separation agreement; Husband was living
with Fiancee and the children at the time the agreement was signed, and his planned marriage to
Fiancee was the reason Husband was willing to sign the agreement as presented. Thus, because
Husbands decision to sign the agreement and accept these support obligations was made with
full knowledge of his impending marriage, he cannot now claim that marriage as a change of
circumstances that warrants modification of his obligations under the divorce decree. Therefore,
the trial court should not modify the spousal-support award based on the needs of Husbands
new wife and her children.

Point Four (25%)


It is unclear whether the trial court erred in refusing to modify the child-support award based on
Husbands nonpaternity of Child.

Traditionally, courts have been extremely reluctant to terminate an established parent-


child relationship and have relied on a wide range of equitable and procedural principles to avoid
doing so. Some courts have found that a prior legal judgment, such as the divorce decree
terminating Husbands and Wifes marriage, bars the introduction of nonpaternity evidence
based on application of res judicata or collateral estoppel principles. See Alan Stephens,
Annotation, Parental Rights of Man Who Is Not Biological or Adoptive Father of Child but Was
Husband or Cohabitant of Mother When Child Was Conceived or Born, 84 A.L.R.4th 655 (1991
& Supp.) (collecting cases). Other courts have relied on the equitable doctrines of estoppel and
laches to achieve the same result. See AM. LAW INST., PRINCIPLES OF THE LAW OF FAMILY
DISSOLUTION: ANALYSIS & RECOMMENDATIONS 3.03 cmt. d (2002); John C. Williams,
Annotation, Laches or Acquiescence as Defense, so as to Bar Recovery of Arrearages of
Permanent Alimony or Child Support, 5 A.L.R.4th 1015 (1981 & Supp.). Moreover, some courts
have simply cited the childs best interest, without relying on any specific legal principle, as
justification either for denying permission to introduce evidence of nonpaternity or for denying a
petition to disestablish paternity. See, e.g., Godin v. Godin, 725 A.2d 904, 911 (Vt. 1998). See
also Stephens, supra. Under this latter approach, the trial court correctly denied Husbands
petition to modify his support obligation based on his nonpaternity because Child is 10 years old,
Husband is the only father Child has known, and there is no evidence that another father-child
relationship could be established.

30
Family Law Analysis

However, in recent years, a number of courts have placed greater emphasis on the
interests of erroneously identified fathers. For example, in M.A.S. v. Mississippi Department of
Human Services, 842 So. 2d 527 (Miss. 2003), the Mississippi Supreme Court held that a
paternity judgment could be vacated more than nine years after its entry based on DNA tests
performed in an unrelated matter which revealed that the petitioner was not the childs biological
father. Although M.A.S. involved a nonmarital child, the Mississippi courts have since applied its
reasoning in a case brought by a divorced petitioner. Although the petitioner had lived with the
child only during its first month of life and had limited contact thereafter, the court did not rely
on the tenuous parent-child relationship in granting the disestablishment petition. Instead, the
court stated its belief that the best interest of the child . . . is to know the identity of the natural
father and refuse[d] to sanction the manifest injustice of forcing a man to support a child which
science has proven not to be his. Williams v. Williams, 843 So. 2d 720, 723 (Miss. 2003). See
also Doran v. Doran, 820 A.2d 1279 (Pa. Super. Ct. 2003) (utilizing similar approach).
Some states have also adopted new paternity disestablishment legislation that places
greater emphasis on the interests of erroneously identified fathers. See, e.g., GA. CODE ANN.
19-7-54 (2009) (mandating paternity disestablishment in certain circumstances when genetic
testing demonstrates a zero percent probability of paternity). See also Paula Roberts, Truth and
Consequences: Part II. Questioning the Paternity of Marital Children, 37 FAM. L.Q. 55, 94
(Appendix F) (2003) (listing 12 state statutes allowing paternity disestablishment by a married
father and noting a strong trend toward enactment of such legislation). Under a legislative or
judicial standard which permits paternity disestablishment without regard to the childs relational
and financial interests, a court could appropriately accept the blood tests offered by Husband,
enter a finding of nonpaternity, and terminate Husbands support obligation.
In a state where the courts or the legislature has adopted rules favoring the ability of
erroneously identified fathers to prove nonpaternity, Husbands motion to disestablish his
paternity and eliminate his child-support obligation would be more likely to succeed. Even in
such states, if Husband was aware of his nonpaternity and had an opportunity to litigate the issue
at the time of entry of the divorce decree, courts are likely to treat the divorce decree as res
judicata on the paternity issue and conclude that there are no changed circumstances warranting a
modification of the child-support order. Nancy Darlene M. v. James Lee M., Jr., 400 S.E.2d 882
(W. Va. 1990). However, some courts have allowed motions to modify a child-support order in a
divorce decree on the basis of nonpaternity, even when that issue was litigated in the original
action, reasoning that a motion to modify is a direct attack on the judgment, not a collateral
attack, and therefore is not barred by res judicata principles. Dixon v. Pouncy, 979 P.2d 520, 524
(Alaska 1999).

31
Contracts Analysis

(Contracts I.A., B.2; IV.A.3; V.A.)

ANALYSIS

Legal Problems: (1)(a) W


as a contract formed when Designer offered Retailer a
discounted payoff in exchange for early cash payment; Retailer
responded, Thanks. Thats a good deal. I dont have the cash to
pay you now. Ill do it if I can get a loan; and Designer replied,
That will be great?

(1)(b) If a contract was formed, did Retailer breach that contract by


making only minimal efforts to obtain the necessary cash?

(2) If Retailer breached the contract, is Retailer liable for Designers


loss caused by Designers inability to make a planned investment
that would have netted a $35,000 gain for Designer?

(3) If Retailer breached the contract, is Retailer liable for punitive


damages?

DISCUSSION

Summary

Retailer and Designer entered into a new, modified contract which Retailer breached.
Designers opening statement was an offer. However, because Retailers response added a
condition, his need to take out a loan to obtain the cash to pay early, the response did not
constitute acceptance of Designers offer. Rather, Retailers response was a counteroffer, which
Designer accepted, thereby creating a contract. That contract contained Retailers implied
promise to make a good-faith effort to borrow the needed cash. Retailer did make some efforts to
get the cash, but these efforts were not sufficient to constitute good faith. Therefore, Retailer
breached the contract by not acting in good faith.
The ordinary measure of damages in a contract action is expectation damages, which aim
to put the nonbreaching party in the position he expected to be in following full performance.
Here, if Retailer had not breached, Designer would have had $15,000 to make an investment that
would have netted a $35,000 gain. Because these losses were foreseeable and reasonably certain,
Designer will be able to recover his actual damages.
Designer will not be able to collect punitive damages because Retailers conduct was not
tortious.

Point One(a) (40%)


Designer made an offer to Retailer to modify their existing contract. Retailer did not accept the
offer. Instead, Retailer made a counteroffer, which Designer then accepted, forming a modified
contract.

32
Contracts Analysis

Designers statement I need cash quickly to make an investment. . . . [I]f you promise
now to pay me $15,000 in cash by the 25th of this month, I will accept that payment as satisfying
your obligation under our contract for this year was an offer to enter into a new, modified
contract. Retailer did not accept this offer. Instead, Retailer said, Thanks. Thats a good deal. I
dont have the cash to pay you now. Ill do it if I can get a loan. This response, impliedly
making Retailers obligation to perform conditioned upon his getting a loan, contained an
additional term and was, therefore, a counteroffer. See RESTATEMENT (SECOND) OF CONTRACTS
59. Designers response, That will be great, indicated assent to the terms of Retailers
counteroffer even though it did not include specific words of acceptance. There was
consideration for the modified contract under the bargained-for-exchange test because Designer
agreed to accept a reduced payment and Retailer agreed to pay early if he could get a loan. See
id. 71.
It might be argued that Retailers response did not constitute a counteroffer because
Retailer did not explicitly state that his performance was conditioned upon getting a loan.
However, Retailers response made it clear that Retailer could not complete the contract unless
Retailer obtained a loan, and Designer certainly understood this. Therefore, the parties entered
into a contract on the terms offered by Retailer.
[NOTE: An examinee may discuss this issue in terms of an amendment to the existing
contract. Such an analysis should receive full credit.]

Point One(b) (30%)


Retailer breached the contract by failing to make good-faith efforts to obtain the necessary loan.

Retailer might argue that because he did not get the loan, his obligation to pay never
arose and he was not in breach. In fact, the language used only suggested that Retailers
obligation to perform would be subject to a condition precedent, that is, his getting the loan.
However, the contract contained an implied obligation to make good-faith efforts to
obtain a loan. Without such an implied obligation, the contract would be illusory because
Retailer would have no obligation. See Wood v. Lucy, Lady Duff-Gordon, 222 N.Y. 88, 118 N.E.
214 (1917); Mezzanotte v. Freeland, 200 S.E.2d 410 (N.C. Ct. App. 1973).
To satisfy the requirement of making good-faith efforts, Retailer must have taken
reasonable steps to obtain the necessary cash. Here, Retailer did no more than pick up two loan
applications which he did not even submit. His actions were not sufficient to constitute a good-
faith effort to obtain the loan. Retailer therefore breached the contract.

Point Two (20%)


Designer is entitled to recover from Retailer his actual damages, which would include the gain
that he would have realized on the investment he intended to make.

The normal measure of damages for breach of contract is expectation damages, which
aim to give the nonbreaching party the benefit of his bargain. See FARNSWORTH, CONTRACTS
12.1 (3d ed. 2004). Expectation damages must be foreseeable, see Hadley v. Baxendale, 156
Eng. Rep. 145 (1854), and proved with reasonable certainty, see FARNSWORTH, supra, 12.1.
Here, when Designer made the initial offer, he stated that he needed cash quickly to make
a potentially profitable investment. On these facts, it is foreseeable that Designer would not be
able to make the investment when Retailer failed to make timely payment. It is not necessary that
the profitability of the investment be foreseeable at the time of the breach. It is enough that the

33
Contracts Analysis

fact of the investment is foreseeable, and here it was, because Designer told Retailer that
Designer needed the cash to make an investment, making the damages foreseeable. The facts
state that Designer can prove that he would have made $35,000 on the investment, making the
damages reasonably certain. Therefore, Retailer is liable for Designers actual damages.

Point Three (10%)


Designer is not entitled to recover punitive damages because such damages are not ordinarily
recoverable in a contract action.

Punitive damages are not generally recoverable as an element of damages in a breach of


contract action unless the conduct constituting the breach is also a tort for which punitive
damages [can be recovered]. RESTATEMENT (SECOND) OF CONTRACTS 355. The facts do not
indicate any conduct on the part of Retailer that would constitute a tort compensable through
punitive damages. Therefore, Designer is not entitled to recover punitive damages.

34
Corporations Analysis

(Corporations and Limited Liability Companies I.A.; II)

ANALYSIS

Legal Problems: (1) When did Deltas corporate existence begin?

(2) Is Adam, Baker, or Clark personally liable on the Mega contract?

(3) Is Adam, Baker, or Clark personally liable on the Sole contract?

DISCUSSION

Summary

Deltas corporate existence began only upon Adams filing of Deltas articles of
incorporation on November 15. The recitation in the articles of a stated earlier effective date
(September 1) is of no legal consequence. Only Adam is personally liable on the Mega contract
because Adam alone participated in entering into the contract with knowledge that the articles
had not been filed. Adam and Baker, but not Clark, are liable on the Sole contract. That contract
was executed by Delta before its articles were filed. Adam and Baker, but not Clark, participated
in entering into the contract with knowledge that Deltas articles had not been filed. Thus, Adam
and Baker are personally liable for the Sole contract, and Clark is not personally liable.

Point One (20%)


Delta achieved corporate status upon the filing of its articles of incorporation on November 15.
The filing of Deltas articles of incorporation does not relate back to the earlier date of
September 1 specified in the articles.

Under the Model Business Corporation Act (MBCA) 2.03(a) (1984, with 2000
amendments), . . . corporate existence begins when the articles of incorporation are filed.
Because [i]ncorporation under modern statutes is so simple and inexpensive . . . nothing short
of filing articles of incorporation should create the privilege of limited liability. Id., ch. 2
official cmt.
This date-of-filing rule applies even if the filed articles recite an effective date prior to the
date of filing. The MBCA allows the parties to specify a delayed effective date but not an
earlier effective date. Id. 2.03(a). Thus, the recitation in Deltas articles of an earlier effective
date is ineffective; Deltas corporate existence began on the date that Adam filed the articles.

Point Two (40%)


When a corporations articles of incorporation have not been filed, persons purporting to act for
the corporation are personally liable for corporate debts if they knew when the debt was incurred
that there was no incorporation.

The fact that Deltas articles of incorporation were not filed when Delta entered into the
contract with Mega does not automatically make Adam, Baker, and Clark liable to Mega for

35
Corporations Analysis

amounts due on that contract. Under the MBCA, [a]ll persons purporting to act as or on behalf
of a corporation, knowing there was no incorporation under this Act, are jointly and severally
liable for all liabilities created while so acting. 2.04. Thus, under 2.04 both the participation
and knowledge requirements must be met to hold any of Adam, Baker, and Clark liable.
With regard to the Mega contract, only Adam purported to act on Deltas behalf and only
Adam knew that Deltas articles had not been filed. Therefore, Adam is personally liable on the
Mega contract. On the other hand, neither Baker nor Clark satisfies either the participation or
knowledge requirement. Thus, neither is liable on the Mega contract.
[NOTE: The facts clearly state when Baker learned that the articles had not been filed:
October 15. Clark, however, at all times believed that Deltas articles of incorporation had been
filed.]
[NOTE: Examinees may discuss the de facto incorporation doctrine under which a
shareholder is not liable for the obligations of a defectively incorporated corporation when there
was a bona fide attempt to organize . . . and colorable or apparent compliance with the
requirements of the law. Gillette v. Aurora Ry. Co., 81 N.E. 1005, 1009 (Ill. 1907); Milligan v.
Milligan, 956 So. 2d 1066, 1074 (Miss. Ct. App. 2007). Such analysis is inapplicable here. First,
the MBCA (which the facts state governs this problem) abolished the doctrine. Even in a non-
MBCA jurisdiction, the doctrine would not apply here because Adam did nothing to attempt
compliance with the statutory requirements. Without allegations that a certificate of formation
was ever prepared or filed, there is no basis . . . to claim a good-faith attempt to comply with
[statutory requirements]. Leber Associates, LLC v. The Entertainment Group Fund, Inc., No. 00
Civ. 3759, 2003 U.S. Dist. LEXIS 13009 at *33 (S.D.N.Y. July 29, 2003) (applying Delaware
law).]

Point Three (40%)


Persons purporting to act for a corporation are personally liable for a corporate contract if they
entered into the contract with knowledge that there was no incorporation.

When a corporations articles of incorporation have not been filed, a person is liable for
pre-incorporation transactions only when the person purporting to act on behalf of a corporation
not yet formed possesses actual knowledge that the corporations charter has not yet been issued.
See MBCA 2.04; Weir v. Kirby Construction Co., Inc., 446 S.E.2d 186 (1994). It is not enough
to establish liability that a person should have inquired about the entitys status and should have
known that the corporation was not formed, as those facts do not rise to the level of actual
knowledge required. See Sivers v. R & F Capital Corp., 858 P.2d 895, 898 (Or. Ct. App. 1993).
Here, with regard to the Sole contract, Adam and Baker knew that Deltas articles of
incorporation had not been filed at the time Delta entered into that contract, while Clark did not
know. Baker might argue that, as a mere prospective shareholder, he was not a [person]
purporting to act as or on behalf of a corporation, MBCA 2.04, and thus has no liability. This
argument is likely to fail. With regard to the Sole contract, Baker was more than a prospective
shareholderhe participated in the negotiation of the contract and therefore purported to act on
behalf of a corporation. Further, the official commentary to 2.04 suggests that its protection
does not extend to individuals in Bakers position. ([Section 2.04] does not foreclose the
possibility that persons who urge defendants to execute contracts in the corporate name knowing
that no steps to incorporate have been taken may be estopped to impose personal liability on
individual defendants.) Rather, the provision is intended to shield those who erroneously but in
good faith believe that the necessary documents have been filed. See Cranson v. IBM, 200 A.2d

36
Corporations Analysis

33 (Md. 1964). Here, at the time of the Sole contract, Baker knew that Deltas articles had not
been filed and therefore did not have the necessary good-faith belief to the contrary. Thus, Baker,
together with Adam, will be personally liable on the Sole contract. Clark, however, is not liable
because he does not meet the participation requirement nor does he satisfy the knowledge
requirement; he believed at all relevant times that the articles had in fact been filed. However,
even if Clark had knowledge that the articles had not been filed, he would not be liable on the
Sole contract because he does not meet the participation requirement.

37
Federal Civil Procedure Analysis

(Federal Civil Procedure I.A.1; IV.A. & C.)

ANALYSIS

Legal Problems: (1) May a defendant make a motion to dismiss for insufficiency of
service of process when the defendant has already moved to
dismiss for failure to state a claim upon which relief can be
granted?

(2) Do the Federal Rules of Civil Procedure permit a defendant to join


a cross-claim against a co-defendant if it arises out of the same
events as the plaintiffs claim against the defendants?

(3)(a) Does a federal district court have independent subject-matter


jurisdiction over a state law cross-claim for $20,000 between two
defendants who are citizens of the same state?

(3)(b) Does a federal district court have supplemental subject-matter


jurisdiction over a state law cross-claim for $20,000 between two
defendants who are citizens of the same state?

DISCUSSION

Summary

Because Bartender made a motion to dismiss the action against him for failure to state a
cause of action without raising his defense of insufficiency of service of process, he was barred
from raising that defense at a later time. The court therefore was correct in denying his motion,
even if Bartender had a valid claim that the manner of service was improper.
Bartenders claim against Dave arises out of the same transaction or occurrence as
Plaintiffs claim. The claim therefore qualifies as a cross-claim under Rule 13(g) of the Federal
Rules of Civil Procedure, and Bartender may state that claim in his answer to the complaint.
A federal court would not have subject-matter jurisdiction over Bartenders claim against
Dave if that claim were filed as an independent action. However, Plaintiffs original claim and
Bartenders cross-claim are sufficiently closely related to form part of the same case or
controversy under Article III of the United States Constitution. Accordingly, the supplemental
jurisdiction statute authorizes the federal court to take jurisdiction over Bartenders cross-claim.

Point One (30%)


The court correctly denied Bartenders motion to dismiss for insufficiency of service of process
because that defense was not raised when Bartender made his motion to dismiss for failure to
state a claim.

Although it is likely that service of process was improper in this case, the district court
nonetheless was correct to deny Bartenders motion to dismiss on that basis. Bartender initially

38
Federal Civil Procedure Analysis

responded to Plaintiffs complaint by making a motion to dismiss for failure to state a cause of
action. FED. R. CIV. P. 12(b)(6). When that motion was denied, Bartender moved to dismiss for
insufficiency of service of process. FED. R. CIV. P. 12(b)(5).
When a motion pursuant to Rule 12(b) is initially made, a party may join with that
motion all other motions permitted by Rule 12 that are then available to the party. If a party
makes a Rule 12(b) motion and fails to join certain other available defenses or objections, the
party may not later raise those objections and defenses. In particular, a party that makes a Rule
12(b) motion but omits from that motion a defense of insufficiency of service of process is
deemed to have waived that defense. FED. R. CIV. P. 12(h)(1).
Because Bartender made a Rule 12(b) motion to dismiss for failure to state a claim but
did not join his available defense of insufficiency of service of process, Bartender waived that
defense. Hence, the district court was correct to deny Bartenders motion to dismiss on that
ground. It does not matter that Bartender appears to have had a valid claim that service was
improper.

Point Two (30%)


Rule 13(g) of the Federal Rules of Civil Procedure permits a defendant, in the answer, to state a
cross-claim against a co-defendant if it arises out of the same transaction or occurrence as the
original claim.

Rule 13(g) of the Federal Rules of Civil Procedure provides that a defendants answer
may state as a cross-claim against a co-defendant any claim that aris[es] out of the transaction
or occurrence that is the subject matter of the original action between the plaintiff and the
defendants. The policy underlying Rule 13(g) is efficiency; multiple lawsuits are avoided if
defendants bring related cross-claims in the context of the plaintiffs suit. 6 CHARLES ALAN
WRIGHT ET AL., FEDERAL PRACTICE AND PROCEDURE CIVIL 1431, at 267 (2010).
Here, Plaintiffs claims concern Daves injury of Plaintiff and Bartenders service of
alcohol to Dave. Bartenders cross-claim concerns Daves injury of Bartender. The question is
whether these various claims arise out of the same transaction or occurrence.
To determine whether two different claims arise out of the same transaction or
occurrence for purposes of Rule 13, federal courts look at a number of factors: (1) whether the
issues of fact and law in the plaintiffs claim and the defendants cross-claim are essentially the
same, (2) whether the same evidence would support or refute the plaintiffs claim and the
defendants cross-claim, (3) whether there is a logical relationship between the plaintiffs claim
and the defendants cross-claim, and (4) whether res judicata would bar a subsequent suit on the
cross-claim. See, e.g., Q Intl Courier, Inc. v. Smoak, 441 F.3d 214, 219 (4th Cir. 2006)
(interpreting the same words in Rule 13(a), the compulsory counterclaim rule); 6 WRIGHT ET AL.,
supra, 1410, 1431 at 277 (stating that the standard for asserting a cross-claim is basically the
same as that for a compulsory counterclaim). The presence of any of these factors supports a
conclusion that the transaction or occurrence requirement is met. See Glass v. IDS Financial
Serv., Inc., 778 F. Supp. 1029, 106162 (D. Minn. 1991); 6 WRIGHT ET AL., supra, 1410 at 58.
The most frequently considered factor is whether there is a logical relationship between
the defendants cross-claim and the plaintiffs original claim. 6 WRIGHT ET AL., supra, 1410 at
61, 65. Here, the alleged batteries of Plaintiff and Bartender occurred in the same place, within
seconds of one another. Bartenders efforts to intercede in the initial battery presumably
prompted Dave to injure Bartender. There appears to be a causal link, and thus a logical
relationship, between the two claims.

39
Federal Civil Procedure Analysis

In addition, the two claims involve almost identical factual and legal issues. Both
Plaintiffs claim against Dave and Bartenders cross-claim against Dave are claims for battery
arising under State As law. Both claims involve factual questions about the cause and course of
the fight in which Dave, Bartender, and Plaintiff were involved. Even though the factual issues
may differwhether Dave battered Plaintiff is a distinct question from whether Dave battered
Bartenderthe substantial overlap between the legal and factual issues supports the conclusion
that the claims arise from the same transaction or occurrence. The identity-of-issues inquiry does
not require a complete overlap between the claim and the cross-claim. Id. 1410, at 59.
The same evidence test is also likely to be satisfied. To prove the battery claim against
Dave, Plaintiff will need the testimony of witnesses to the brawl. It is likely that the patrons of
the bar who witnessed Daves alleged battery of Plaintiff also witnessed Daves alleged battery
of Bartender. Thus, a substantial amount of the evidence (i.e., the testimony of the witnesses to
the event) will be the same.
The only factor that militates against allowing Bartender to bring a cross-claim is that a
judgment on Plaintiffs claim against Dave would not preclude Bartender, who is not in privity
with either Dave or Plaintiff, from suing on his own claim. Denying Bartender the ability to
bring the claim would not prevent him from having his case heard at some future point. Id.
1431, at 27576. However, given that Bartenders claim against Dave is logically related to
Plaintiffs claims, that it raises overlapping factual and legal issues, and that the same evidence is
relevant to both claims, a court should allow Bartenders claim to be included as a cross-claim in
Bartenders answer.

Point Three(a) (20%)


A federal district court would not have independent subject-matter jurisdiction over a state law
cross-claim for $20,000 between two defendants who are citizens of the same state because the
claim does not arise under federal law, the claimants are not diverse, and the amount-in-
controversy requirement for diversity cases is not satisfied.

The federal question statute grants the federal district courts original jurisdiction of all
civil actions arising under the Constitution, laws, or treaties of the United States. 28 U.S.C.
1331. A claim arises under federal law if federal law creates the cause of action. Merrell Dow
Pharms. Inc. v. Thompson, 478 U.S. 804, 808 (1986); Am. Well Works Co. v. Layne & Bowler
Co., 241 U.S. 257, 260 (1916) (Holmes, J.) (A suit arises under the law [that] creates the cause
of action.). Here, state law creates Bartenders battery claim against Dave. Under this test,
Bartenders cross-claim does not arise under federal law, and therefore the court would not have
jurisdiction under the federal question statute, 28 U.S.C. 1331.
Nor would a federal court have diversity jurisdiction over Bartenders claim. First, there
is no diversity. Bartender and Dave are both citizens of State A. Second, Bartender is seeking
damages of only $20,000, well below the $75,000 jurisdictional minimum in diversity actions.
See 28 U.S.C. 1332(a)(1).

Point Three(b) (20%)


A federal district court would have supplemental subject-matter jurisdiction over a state law
cross-claim for $20,000 between citizens of the same state as long as it is so related to the
plaintiffs claim against the defendants, of which the court has original jurisdiction, that the
claims form part of the same case or controversy under Article III of the Constitution.

40
Federal Civil Procedure Analysis

In the instant case, the federal district court has original diversity jurisdiction over
Plaintiffs claims against Bartender and Dave. 28 U.S.C. 1332. Each claim is for an amount in
excess of $75,000, and Plaintiff, a citizen of State B, is diverse from both defendants, who are
citizens of State A.
The supplemental jurisdiction statute permits federal district courts that have original
jurisdiction of a civil action to assert supplemental jurisdiction over all other claims that are so
related to claims in the action within such original jurisdiction that they form part of the same
case or controversy under Article III of the United States Constitution. 28 U.S.C. 1367(a).
Claims form part of the same case or controversy under Article III if they derive from a
common nucleus of operative fact. United Mine Workers of America v. Gibbs, 383 U.S. 715,
725 (1966). Cross-claims that satisfy Rule 13(g)s transaction or occurrence test presumptively
satisfy the Gibbs common nucleus of operative fact test and are within the courts
supplemental jurisdiction. 13 WRIGHT ET AL., supra, 3523, at 16573. As noted in Point One,
Bartenders claim and Plaintiffs claims arise out of the same occurrence, raise many of the same
factual and legal issues, and will be proved, in part, by the same evidence. These claims clearly
arise out of a common nucleus of operative factthe bar fight that gave rise to all the claims
in the case.
Because Bartenders claim is part of the same case or controversy as Plaintiffs original
claims, it falls within the federal courts supplemental jurisdiction.

41
Decedents Estates Analysis

(Decedents Estates II.A.4., D., I.1. & 9., K.4.)

ANALYSIS

Legal Problems: (1) Was Testators will validly executed?

(2) Should the life insurance proceeds be distributed to Niece or


Cousin?

(3) Does the handwritten memorandum found in Testators safe-


deposit box determine the disposition of Testators jewelry?

(4) Did the bequest to Husband lapse?

(5) How should the general bequests in Testators will abate?

DISCUSSION

Summary

Testators will is valid and should be admitted to probate because Testator complied
with the statutory requirements in executing the will. Testators life insurance proceeds will
probably go to Niece because life insurance contracts almost always disallow alteration of a
beneficiary designation in a life insurance contract by will, and most states have upheld such
contractual requirements. Testators jewelry will probably go to Niece because the memorandum
describing the gift to Niece preexisted Testators will and thus was incorporated by reference
into Testators will. The gift of $40,000 to Husband lapsed because he predeceased Testator, and
bequests to spouses are not typically saved from lapse by an antilapse statute. It is irrelevant that
Husband had a child who survived Testator. The general bequests of $25,000 each to Church,
Library, and School must be abated because there are insufficient funds ($60,000) to pay these
bequests. Because there are no indications in the will specifying a different abatement scheme,
Church, Library, and School will take equal shares (i.e., $20,000) of Testators bank account. As
a result of the abatement, nothing remains in the estate to pass to Son. Niece need not contribute
to those general bequests because life insurance proceeds are nonprobate assets not governed by
the will. Furthermore, the devise of items listed in a memorandum was specific, and specific
devises are the last to abate.

Point One (20%)


Because Testator complied with the statutory execution requirements, her will is valid.

Here, state law disallows all holographic wills and codicils. A holographic will is an
unwitnessed will in a testators handwriting. Testators will was not holographic even though
written in the testators hand because it was signed and witnessed.

42
Decedents Estates Analysis

State law does provide that a will is not valid unless it is acknowledged by the testator to
the witnesses and signed by the testator in the presence of at least two attesting witnesses, who
shall sign their names below that of the testator within 30 days. Testator signed her will in the
presence of three witnesses who, on the same day, signed their names below hers. Testator
acknowledged the instrument to be her will when she stated, This is my will. I would like each
of you to witness it. Therefore, Testators will is valid because its execution complied with the
law of the state.

Point Two (20%)


Niece will probably take Testators life insurance because, in most states, Testator could not
change her life insurance beneficiary except as specified in her life insurance contract.

If an insured dies while a life insurance policy is in effect, the policy proceeds are
payable to the named beneficiary. Although the owner of a life insurance policy typically retains
the right to change the named beneficiary without obtaining that beneficiarys consent, the owner
must do so in accordance with procedures specified in the life insurance contract.
Life insurance contracts almost never permit a change of beneficiary by will, and courts
have almost invariably upheld such restrictions. See ROBERT J. LYNN & GRAYSON M.P.
MCCOUCH, INTRODUCTION TO ESTATE PLANNING IN A NUTSHELL 13132 (5th ed. 2004); Cook v.
Equitable Life Assurance Society, 428 N.E.2d 110 (Ind. Ct. App. 1981). Courts have sometimes
found that a beneficiary change that does not comply with the terms of the insurance contract is
valid if the policyholder has substantially complied with the contract by taking all reasonable
steps within his or her power to make the change in accordance with the contract terms. See
LYNN & MCCOUCH, supra, at 13132. Courts in a few states have also upheld a change of
beneficiary by will. See, e.g., Burkett v. Mott, 733 P.2d 673 (Ariz. Ct. App. 1986) (relying on
insurance companys failure to object); Connecticut Gen. Life Ins. Co. v. Peterson, 442 F. Supp.
533 (W.D. Mo. 1978).
Here, Testator attempted to change the beneficiary of her policy by naming Cousin as the
beneficiary in her will. Assuming that Testator had a standard life insurance contract, such a
change would be disallowed. There is nothing in the facts to show that Testator made any
attempt to comply with the formalities required by the insurance contract. Thus, unless this state
follows the minority approach, Niece, the named beneficiary in the policy, will take the policy
proceeds.

Point Three (20%)


Niece will probably take Testators jewelry because, in most states, the memorandum describing
the bequest was incorporated by reference into Testators will.

In most jurisdictions, a writing that exists at the time a will is executed may be
incorporated by reference into the will. The will must describe the writing with sufficient
particularity that it can be identified, but the writing need not be witnessed or executed with
testamentary formalities. See LYNN & MCCOUCH, supra, at 18384; UNIF. PROBATE CODE
2-510.
Here, the memorandum describing the bequest of jewelry to Niece is clearly identified in
Testators will, and it was found in the location described by Testator. The memorandum is dated
on the day before Testators will was executed, so it qualifies as a document that can be
incorporated by reference. Thus, assuming that this state follows the approach of the

43
Decedents Estates Analysis

overwhelming majority of states and allows incorporation by reference, Niece will take the
jewelry.
In addition, some states recognize the right of a testator to dispose of tangible personal
property by a signed memorandum, whether prepared before or after the execution of the will,
even though it is not executed with the formalities required of wills. See, e.g, Iowa Code
633.276; UNIF. PROBATE CODE 2-513.
[NOTE: The difference between UPC 2-513 and the incorporation-by-reference
doctrine is that the former allows a testator to dispose of tangible personal property by a signed
memorandum executed both before and after the will was signed. Under the incorporation-by-
reference doctrine, the memorandum must be in existence at the time the will is executed.]

Point Four (20%)


The bequest to Husband lapsed because he predeceased Testator. Under the typical antilapse
statute, the bequest to Husband would not be saved from lapse even though he left issue who
survived Testator.

If a beneficiary does not survive the testator, the bequest lapses and falls into the residue
of the estate. Under the common law, lapse occurred whenever a beneficiary predeceased the
testator and the testator specified no alternate disposition of the assets in question. A lapsed
bequest passes into the residue of the testators estate. See WILLIAM M. MCGOVERN, JR. &
SHELDON F. KURTZ, WILLS, TRUSTS AND ESTATES 32829 (3d ed. 2001).
Today, all states have antilapse statutes. Under a typical antilapse statute, if a beneficiary
dies before the testator and the beneficiary was both related by blood to the testator within a
certain degree of relationship and had issue who survived the testator, the bequest to the
deceased beneficiary is saved from lapse and the deceased beneficiarys issue takes in lieu of the
deceased beneficiary. See, e.g., UNIF. PROBATE CODE 2-603.
The typical antilapse statute, therefore, would not save a bequest to a spouse even if the
spouse predeceased the testator leaving issue who survived the testator. Id.
Thus, the $40,000 bequest to Husband lapses and passes into the residuary estate.
[NOTE: In New Hampshire and Kansas the result differs, because their antilapse statutes
would apply to Husband. In those states, his share would pass to his daughter.]

Point Five (20%)


Testators general legacies must be abated because there are insufficient assets to pay all named
beneficiaries. Under standard abatement principles, Son will receive nothing and Library,
Church, and School will receive equal, pro rata general bequests. Niece will take the jewelry
because specific devises abate last. She also will take the life insurance because it passes outside
the will.

When the assets of a testators estate are insufficient to pay all of the bequests payable
under the testators will, these bequests are reduced, or abated. Under the common law and in
most states, unless the testator specifies a different abatement scheme, testamentary bequests
abate in the following order: (1) residuary bequests, (2) general bequests (i.e., bequests of a fixed
dollar amount), and (3) specific bequests (i.e., identifiable property such as my jewelry).
Abatement within each category is pro rata. See UNIF. PROBATE CODE 3-902; LYNN &
MCCOUCH, supra, at 4748.

44
Decedents Estates Analysis

Here, there are insufficient assets in Testators estate to pay the bequests made under
her will, and Testator specified no abatement scheme. Thus, the residuary bequest to Son abates
first; Son will receive nothing. The general bequests to Church, Library, and School, totaling
$75,000, abate next and pro rata as each of them was bequeathed $25,000. Thus, Church,
Library, and School will each receive $20,000 from the available $60,000. Niece need not
contribute because the bequest of jewelry is a specific devise and, assuming this state follows the
majority view, the life insurance is a nonprobate asset that will not be taken into account when
abating the testamentary bequests. See UNIF. PROBATE CODE 3-902.
[NOTE: If Husbands daughter is entitled to the $40,000 bequest to Husband, there will
be $115,000 of general legacies to abate proportionally when distributing the $60,000.]

45

Anda mungkin juga menyukai